LSAT and Law School Admissions Forum

Get expert LSAT preparation and law school admissions advice from PowerScore Test Preparation.

 Administrator
PowerScore Staff
  • PowerScore Staff
  • Posts: 8917
  • Joined: Feb 02, 2011
|
#22762
Complete Question Explanation

Must Be True. The correct answer choice is (C)

The stimulus in this case offers a series of conditional reasoning statements, which can be diagrammed as follows:
  • (1) Lower Revenues → Changed Attitudes or Not Affordable

    (2) Changed Attitudes → Something to Celebrate

    (3) Not Affordable → Salaries Kept Pace
The question stem instructs us to assume that these are all true and then gives us this:

Salaries Kept Pace

The only place in the premises that we see this is in statement (3). Taking the contra-positive, we see:
  • Not Affordable → Salaries Kept Pace

    Salaries Kept Pace→ Affordable

Since salaries have kept pace, we know that the goods were affordable. This will likely be an important inference when we are searching for the right answer.

Answer Choice (A) is tricky, but wrong: since goods are affordable, if there are lower revenues, it must have happened because attitudes have changed. But, we do not know if there are lower revenues, so we cannot say anything about shoppers’ attitudes.

Answer choice (B) is the opposite of (A): it assumes that attitudes have not changed and then asserts that revenues have not gone down. But, we have no information about whether attitudes have actually changed or not.

Answer choice (C): This is the correct answer choice, since it repeats the inference we were able to draw from the stimulus.

Answer choice (D) makes the same mistake as (B): we do not know whether attitudes have changed.

Answer choice (E) is similar to (D). Since we know that prices have not risen beyond an affordable level (this was given in the question stem), (E) is just an assertion that attitudes have change. But, again, we do not know this for a fact.
 ellenb
  • Posts: 260
  • Joined: Oct 22, 2012
|
#8826
Dear PowerScore,

I just had a question in regards to the diagram for this question:
Attitude Changes--> Celebrate Season
Decreased Revenues--> or
Not Affordable-->Not Kept Pace

Than we can have the CP:

not Celebrate Season-->No Attitude Changes
AND ----> not Decreased Revenues
Kept Pace---> Affordable

It seems like they did not do a CP in the explanations, is this CP correct? based on it we can actually deduce B if kept pace--->affordabe--> not decreased revenues.

However, I was debating between B and C, I know that C is correct, but just want to make sure that I am making a valid Cp

thus if I had

A---> B--> D
or
C--> E
Than the Cp will be:


not E-->not B
and -->not A
not D-->not C


Please let me know if I have the origial plus the cp correct, and why B is not the correct answer choice.

Thanks in advance

Ellen
 Steve Stein
PowerScore Staff
  • PowerScore Staff
  • Posts: 1153
  • Joined: Apr 11, 2011
|
#8827
Hi Ellen,

This stimulus provides another bunch of conditionals:
..... ..... ..... ..... ..... change in attitude ..... ..... :arrow: celebrate
Decrease in revenues :arrow: ..... or
..... ..... ..... ..... ..... prices risen to unaffordable :arrow: salaries NOT kept pace

Contrapositives:

NOT Celebrate ..... :arrow: NOT change in attitude

Salaries kept pace :arrow: Prices NOT risen to unaffordable (correct answer)

For clarity, I am not including the above two contrapositives in the AND portion of the diagram:

NOT change in attitude
..... AND ..... ..... ..... :arrow: NO decrease in revenues
Prices NOT risen to unaffordable

Note that in the diagram directly above, we need to know BOTH sufficient conditions to conclude that there has been no decrease in revenues. All we can actually confirm is the part about prices--based on the information in the stimulus, there is no way to tell whether or not there has been a change in attitude.

I hope that's helpful! Please let me know whether that's clear.

~Steve
 ellenb
  • Posts: 260
  • Joined: Oct 22, 2012
|
#8858
I got it basically when we have,


..... B --->N
A--> or
..... C---->P

and we know Not P we can conclude only not C, since to conclude not A, we need to have either not N or not B, correct?

How about if I have:

..... B --->N
A--> and
..... C---->P

and I know that not P, than can I conclude not A?

Not P-->Not C--->Not A

Regards,

Ellen
 ellenb
  • Posts: 260
  • Joined: Oct 22, 2012
|
#9251
Thanks every bit of concept understanding counts!!! Getting closer and closer to full understanding. Thanks Dave, every drop of understanding counts!
 Steve Stein
PowerScore Staff
  • PowerScore Staff
  • Posts: 1153
  • Joined: Apr 11, 2011
|
#9293
Hi Ellen,

Thanks for your kind words--glad we could be of assistance!

~Steve
 ssalves
  • Posts: 1
  • Joined: Oct 07, 2015
|
#20147
Hi Steve,

Why is the answer not A? I can see why the answer is C very clearly, but still, I do not know how I can rule out A.

Here's what we have: If Decrease in revenues then either change in attitude OR prices risen to unaffordable .

Because this is an Either/Or relationship, I wrote;

NO change in attitude --> prices risen to unaffordable
[as well as]
NO prices risen to unaffordable --> change in attitude.

So if we showed that answer c) NO Prices risen to unaffordable, why can't we conclude that A) change in attitude?

I am asking because I recently read your Either/Or discussion in the LR Bible.
Question 2 about the Editorialist at the end of Ch 6 in the LR Bible is very similar, and the answer had a very similar method to the one I just described.

The only difference I can note is that the current problem about the retail store has a MBT question stem, and the problem in The LR Bible is a most strongly supported stem, but I'm still not sure what that means (that is, it still seems to me that A would follow). Can you help please?

Thank you,
Sandra
 Steve Stein
PowerScore Staff
  • PowerScore Staff
  • Posts: 1153
  • Joined: Apr 11, 2011
|
#20149
Hi Sandra,

Great question! And it's great that you're on the lookout for relevant conditional relationships to diagram. Take another look at the first sentence: "If retail stores experience a decrease in revenues during this holiday season..." This means that "decrease in revenues" would be the sufficient condition— the condition that would be sufficient to conclude that either attitudes have changed or prices have risen.

The issue, though, is that neither the author nor answer choice (C) provides that there will be such a decrease in holiday revenues this year, so there is no basis to conclude that the either/or statement would hold true.

The example that you referred to in the Logical Reasoning Bible is a great one; similarly, it begins with a sufficient condition ("Drivers with a lot of points who have been convicted"). In that example, the correct answer choice begins by providing that sufficient condition.

Again, great question—I hope this is helpful—please let me know whether this is clear—thanks!

~Steve
 Johnclem
  • Posts: 122
  • Joined: Dec 31, 2015
|
#26583
I just had a quick question with this one.
Would it be wrong to connect all the conditional statements into one change in this one ?

Decrease in rev --> attitude change --> celebrate
Or
Price risen --> salaries not kept in pace


Thanks
John

Get the most out of your LSAT Prep Plus subscription.

Analyze and track your performance with our Testing and Analytics Package.